LSAT and Law School Admissions Forum

Get expert LSAT preparation and law school admissions advice from PowerScore Test Preparation.

 donger
  • Posts: 21
  • Joined: Jun 28, 2012
|
#13953
Dear Powerscore,

I'm a student who's purchased the Powerscore LR and LG Bibles and took 10 hours of Powerscore tutoring.

I had a question about answer choice B in the relevant question and did not understand how that answer choice weakens the argument. When I first read the stimulus, my head automatically scanned through answer choices for one of the 5 Cause and Effect weakening scenarios (i.e. find alternate causes). I looked to eliminate answer choices that showed that there was an increase in the number of claims for reasons other than self-serving motivations. That way I eliminated several answers.

I eliminated C because I thought that if C was true, then that would mean that the workers who filed claims in the stimulus actually did suffer injuries and thus had reasons that were not purely self-serving. I thought this was in line with the nature of Third Family question types.

B, on the other hand, does not explain the sharp increase in number of claims. If employees were dismissed once the claims were filed, then the claims have already been filed and do not contribute to a sharp increase in claims filed AFTER the factory closed.

The length of my post may have given it away but I have a tough time understanding this question. Please help!

Donger
 Adam Tyson
PowerScore Staff
  • PowerScore Staff
  • Posts: 5153
  • Joined: Apr 14, 2011
|
#13960
Hey Donger, another good question here. Your analysis looks pretty good - you correctly identified this as a weaken-except question, and you also correctly looked for the classic cause and effect answer choices in order to eliminate them. If an answer gives an alternate cause (other than the desire to gain undeserved benefits that the author imputes to those who filed claims after the factory closed), then it weakens and is an incorrect answer.

Answer B provides such an alternate cause - workers may have been motivated to delay filing their claims during the term of their employment for fear that they would be fired, since there was a pattern of that happening. Once the factory closed, there was no need to fear that anymore, so they went ahead and filed. They were motivated not by a desire to get away with something, but by freedom from fear of losing their jobs.

Answer C does not weaken the argument - if anything, it strengthens it. If most people file on the day they are injured, why did these people wait? There's something fishy about their decision to wait. Maybe it's because they weren't really injured and just want to gain a benefit that they don't deserve? In any event, the fact that most people file on the day of the injury does not weaken this author's argument about the people that delayed filing.

I hope that helped, at least by making clear why B does weaken the argument and therefore is incorrect. Keep on practicing - it gets easier the more you do it!
 donger
  • Posts: 21
  • Joined: Jun 28, 2012
|
#13962
Oh snap. I didn't see that part of your analysis about B. Thank you.

I just re-read the stimulus for question 22 and I see how C does not work. The stimulus states "AFTER the factory closed," the number of claims increased.

Also, after I review my LR sections, I try to write down general tips that will help me avoid a mistake and one of the tips/guidelines I wrote for weaken questions was: "if I have to layer a condition over my answer choice to make the answer choice correct, then that answer choice is likely incorrect." I would appreciate your opinion on this tip. So in this question, although admittedly I didn't see this layer, I didn't attempt to think beyond what was given in the answer choice. In this question, the layer is the "fear" that could be induced from the factory's pattern and this layer then weakens the answer choice by providing an alternative motive. So had I discerned this layer in a weaken question (not a weaken except question) I might have considered it a loser. Another layer example may be an answer choice that states "He studies hard" in a weakening question with the following conclusion: "He will do poorly on the LSAT." The layer in this quick example would be "because he studies hard, he will succeed." I would thus strike this answer off as a loser. I guess the layer is analogous to the Supporter role in Assumption questions. Do you think I should continue to apply this guideline?

Donger
 Robert Carroll
PowerScore Staff
  • PowerScore Staff
  • Posts: 1787
  • Joined: Dec 06, 2013
|
#14098
Donger,

Although this is a Weaken-Except question, since we're speaking in general about correct answers for the Weaken question type, I'll restrict myself to the normal situation - there is one correct answer choice that Weakens.

Remember that, in order to weaken the argument, the correct answer choice does not need to refute the conclusion entirely. It simply needs to make the conclusion less likely, by any amount. If there is any likelihood that studying hard will lead to success, then the person studying hard is less likely to do poorly on the LSAT. So in your hypothetical, we don't need to think that everyone who studies hard will be successful, just that it has some positive chance of creating success.

Returning to the actual LR question, a reasonable person working for this factory would change their behavior if filing an injury claim led to being dismissed in the past. So we're not adding a "layer" onto answer choice (B), just considering the expected consequences of such a pattern - workers would fear to file a claim. Once that fear is gone, like when the factory closed, there would be no reason to be afraid. So (B) weakens this argument.

I hope this has helped.

Robert
 Frank
  • Posts: 16
  • Joined: Apr 30, 2014
|
#14683
Hey Lucas,

Can you help with this question?

#22 Letter to the editor: After Baerton's factory closed, there was a sharp increase in the number of claims filed for job related injury compensation by the factory's former employees...

I didn't end up picking C but now that I am reviewing it I think I understand that C strengths the authors conclusion. I'm just confused how the other's weaken. I'd kinda like to see how the question is broken down because it took me a while to work through this and I still didn't get it.


Frank
 Adam Tyson
PowerScore Staff
  • PowerScore Staff
  • Posts: 5153
  • Joined: Apr 14, 2011
|
#14686
Hey Frank, I can help here.

As with any weaken question, start with the conclusion. Here, the author is claiming that the people who filed claims after the factory closed did so for essentially dishonest reasons. To weaken that conclusion you want to look for answers that show that the claimants had some good reason to file when they did, other than the dishonest reasons suggested by the author.

Answer A gives one good reason - they had to wait until they were out of work because of the rules around filing such claims. Not dishonest, just following the rules.

Answer B gives another - workers may have feared retaliation from the boss, but that fear went away when the factory closed. Can't get fired from a job you don't have any more! Not dishonest, just no longer afraid.

Answer D suggests that they may have held off filing the claims sooner, while they were working, because there was no point - they would actually lose money going on disability, so they preferred to work despite their injuries. Again, now that the factory is closed there is no reason to wait any more - they may as well file, because getting those benefits is better than getting nothing. Not dishonest, just doing a different cost/benefit analysis in light of their new situations.

Answer E takes a different approach - maybe the sharp increase in claims just after the factory closed was due to a sharp increase in injuries just before it closed, as depressed workers, knowing the factory was going to close soon, became more prone to injuries. Again, no dishonesty on their part - they just hadn't been injured until very recently.

Make the conclusion less likely - in this case, by suggesting an alternate reason (alternate cause?) for the increased claims - and that weakens. Since this is a weaken-except question, four answers weaken and are therefore the wrong answers. The one that doesn't weaken (and it need not strengthen, just NOT weaken) is your winner.

Hope that helped!
User avatar
 CristinaCP
  • Posts: 28
  • Joined: Sep 17, 2023
|
#105243
I understand why C is right, but have a question about why E is wrong. For it to weaken, don't we have to assume that the workers knew they were going to be laid off? We don't know that from the stimulus...
User avatar
 Chandler H
PowerScore Staff
  • PowerScore Staff
  • Posts: 58
  • Joined: Feb 09, 2024
|
#105252
Hi Cristina,

Good question. The conclusion argues that the workers who filed for compensation "were JUST out to gain benefits they did not deserve, and filed ONLY to help them weather their job loss." (Emphases mine.)

Since the conclusion stresses that the ONLY reason the workers filed was to gain undeserved benefits, any answer choice that provides some alternate reason to file will weaken the argument *to some degree*.

Answer choice (E) does provide that alternate reason. You're right that (E) asks us to assume that the workers knew they would soon be laid off, so it's not the best weakener here... but it does weaken the argument *to some degree*, which is what's important in terms of finding the correct answer (i.e., the one that doesn't weaken the argument at all).

Also, for what it's worth, the stimulus likely intends for us to assume that the workers did know they were going to be laid off. If an entire factory closes, it seems very likely that the workers would know in advance that the factory was going to shut down.

Get the most out of your LSAT Prep Plus subscription.

Analyze and track your performance with our Testing and Analytics Package.